Titu Andreescu 102 Combinatorics Advanced
Titu Andreescu 102 Combinatorics Advanced
- 1
|p(l) - 1| + \p{2) - 2| + •••+ |p( n ) n\ =
ever get m ail on the same day, but th at there are never more than
two houses in a row th a t get no mail on the same day. How many
different patterns of m ail delivery are possible?
8 . For i = 1, 2, . . ., 11, let М г- be a set of five elements, and assume
th at for every 1 < i < j < 11, Mj П M j ф 0. Let m be the largest
number for whieh there exist M q , . . ., M Jm among the ehosen
sets with n^= 1 M jfe ф 0. Find the m inim um value of m over all
possible initial ehoiees of М г.
9. Define a d o m m o to be an ordered pair of d istinet positive integers.
A p r o p e r se q u e n c e of dominos is a list of distinet dominos in whieh
the first eoordinate of eaeh pair after the first equals the seeond
eoordinate of the im m ediately preeeding pair, and in whieh ( i , j )
and ( j, i) do not both appear for any i and j . Let D 4 0 be the
set of all dominos whose eoordinates are no larger than 40. Find
the length of the longest proper sequence of dominos th a t ean be
formed using the dominos of D 4 0 .
10. Find the number of subsets of {1 , . . ., 2000 }, the sum of whose
elements is divisible by 5.
11. Let X be a finite set of positive integers and A a subset of X .
Prove th a t there exists a subset B of X sueh th at A equals the
set of elements of X whieh divide an odd number of elements of
B.
12. A staek of 2000 eards is labeled with the integers from 1 to 2000,
with different integers on different eards. T h e eards in the staek
are not in numerieal order. T h e top eard is removed from the
staek and plaeed on the table, and the next eard in the staek is
moved to the b o tto m of the staek. T h e new top eard is removed
from the staek and plaeed on the table, to the right of the eard
already there, and the next eard in the staek is moved to the
b o tto m of the staek. T h is proeess— plaeing the top eard to the
right of the eards already on the table and moving the next eard
in the staek to the b o tto m of the staek— is repeated until all eards
are on the table. It is found th a t, reading left to right, the labels
on the eards are now in aseending order: 1 , 2 , 3 , . . . , 1999, 2000.
In the original staek of eards, how m any eards were above the
eard labeled 1999?
13. Form a 2000 x 2002 sereen with unit sereens. Initially, there are
more than 1999 x 2001 unit sereens whieh are on. In any 2 x 2
14 Advanced Problems
n
2
Advanced Problems 15
a n-1 + 3 a n —2
ai .
19. For a set A, let \A\ and s(A ) denote the number of the elements
in A and the sum of elements in A, respectively. (If A = 0, then
|A| = s(A) = 0.) Let S be a set of positive integers sueh th at
(a) there are two numbers x , y E S with gcd(a?, y) = 1;
(b) for any two numbers x , y E S, x + y E S .
Let T be the set of all positive integers not in S. Prove th at
s ( T ) < \T\2 < oo.
20. In a forest eaeh of 9 animals lives in its own cave, and there
is exactly one separate path between any two of these caves.
Before the eleetion for Forest Gum p, King of the Forest, some of
the animals make an eleetion eampaign. Eaeh eampaign-making
anim al— TÇC (Forest Gum p eandidate)— visits eaeh of the other
caves exactly onee, uses only the paths for moving from cave to
cave, never turns from one path to another between the caves,
and returns to its own cave at the end of the eampaign. It is also
known th at no path between two caves is used by more than one
TÇC. Find the m axim um possible number of T Ç C Js.
21. For a sequence A i , . . ., A n of subsets of {1 , . . ., n ] and a permu-
tation T r o f j S ^ l l j . - . j n j j W e define the diagonal set
D n { A i , A 2 , . . . , A n ) — {г E S |i ÇjL
arithm etie sequence and B does not eontain any infinite arith-
metie sequence.
25. Çonsider the set T 5 of 5-digit positive integers whose deeimal
representations are perm utations of the digits 1, 2, 3, 4, 5.
Determ ine if it is possible to partition T 5 into sets A and B sueh
th at the sum of the squares of the elements in A is equal to the
eorresponding sum for B .
26. Let n b e a positive integer. Find the number of polynomials P ( x )
with eoeffieients in {0 , 1 , 2 , 3 } sueh th at T ( 2 ) = n.
27. Let n and k be positive integers sueh th a t < k < |u. Find
the least number m for whieh it is possible to plaee eaeh of m
pawns on a square of an n x n ehessboard so th at no eolumn or
row eontains a bloek of k adjaeent unoeeupied squares.
28. In a soeeer tournam ent, eaeh team plays eaeh other team exactly
onee and receives 3 points for a win, 1 point for a draw, and
0 points for a loss. After the tournament, it is observed th at
there is a team whieh has b oth earned the most to tal points and
won the f e w e s t games. Find the smallest number of team s in the
tournament for whieh this is possible.
29. Let cq, . . . , a n be the ürst row of a triangular array with a* E
{0 , 1 }. Fill in the seeond row b 1 , . . . , 6 n - 1 aeeording to the rule
bk = 1 if etk ф a k + 1 ) — 0 if a k — a k + 1 - Fill in the remaining
rows similarly. Determ ine with proof the m axim um possible
number of l ’s in the resulting array.
30. There are 10 eities in the Fatlan d. Two airlines eontrol all of
the lLights between the eities. Eaeh pair of eities is eonneeted by
exactly one ffight (in both direetions). Prove th at one airline ean
provide two traveling eyeles with eaeh eyele passing through an
odd number of eities and with no eommon eities shared by the
two eyeles.
31. Suppose th a t eaeh positive integer not greater than n ( n 2 — 2n +
3)/2, n > 2, is eolored one of two eolors (red or blue). Show th at
there must be a m onoehrom atie n-term sequence a\ < a -2 < * ••<
a n satisfying
k- 1
1 < \ S n { * , 2 + 1, г + 2 } | < 2
i GS i- 1
A.n_(_1 — { x + 1 |x C B n } , B n+i — A n U B n An П B n,
for all p o sitiv e in teg ers n . D e te r m in e all th e p o sitiv e in tegers n
sueh t h a t B n = { 0 } .
| 4 U 4 U d i 3 U d i 4| < n - 2 .
P ro v e t h a t k < 2 n ~ 2 .
Solutions to Advanced
Problems
Solutions to Advanced Problems 61
whieh is equivalent to
n 2 — 41 n + 400 = 0.
Solution: We have
= ±1 ± 1 + 2 + 2 ± ---± n± n.
and
n + 3' 'n + 5 72+1
P ,P ,p(n) ^ ^ 1, 2 ,
n —1 n+ 1 n+ 3
,77 {*}
and
77—1
•••>P(n ) f = 1
n —1 72+1 72—1 77 — 1
+ — 77
sueh permutations.
3. [AIME 1986] In a sequence of eoin tosses one ean keep a reeord
of the number of instanees when a tail is immediately followed
by a head, a head is immediately followed by a head, ete. We
denote these by T H , H H , ete. For example, in the sequence
H H T T H H H H T H H T T T T of 15 eoin tosses we observe that
there are hve H H , three H T , two T H , and four T T subse-
quences. How many different sequences of 15 eoin tosses will
eontain exactly two H H , three H T , four T H and five T T
subsequences?
( 1)
( a, . . . , a, a+l, . . . , a+l, . . . , a + s - 1 , . . . , a + s - 1 )
4, 4,
Henee our assumption was wrong and all 23 people must have
equal weights.
6. [IMO Shortlist 1998] Determine the smallest integer n, n > 4, for
whieh one ean ehoose four different numbers a ,b ,c ,d from any n
distinet integers sueh that a + b —e — d is divisible by 2 0 .
{ 0, 20. 40, 1, 2, 4, 7, 12 },
adjaeent houses ever get mail on the same day, but that there are
never more than two houses in a row that get no mail on the same
day. How many different patterns of mail delivery are possible?
n(x) = 55.
x£X
Sinee Мг П Mj ф 0, there are (^1) = 55 nonempty interseetions.
On the other hand, eaeh element x appears in (n interseetions.
Therefore,
It follows that
n ( x ) ( n ( x ) - 1 ) ^ 55
x£X ^
implying that
m —1 ^\
— — 2^ > 55.
x£X
Flenee ^1" 1 > 1, or m > 3. Ifm = 3, then all equalities hold; more
preeisely, n(x) = m = 3 for all an But sinee n(x ) — 55 and
55 is not divisible by 3, n(x) eannot always equal 3. Therefore
m > 4.
Now we prove that m = 4 ean be obtained. We eonsider the
following 4 x 4 array:
a b e d
e / g h
1 2 3 4
5 6 7 8
■ S ^ £/(?)■
j' = i
Note that ^2, ^4, ^5 = 1 are the roots of g(x) = ж5 —1, that
is
It follows that
9 ( ~ 1) = - 2 = ( - 1 - 0 ( - 1 - - Л ( - 1 - С4 ) ( - 1 - е 5 )-
Therefore
( i + 0 ( i + e 2) ( i + e 3 ) ( i + e 4 ) ( i + e 5) = 2
removed from the staek and plaeed on the table, and the next
eard in the staek is moved to the bottom of the staek. The new
top eard is removed from the staek and plaeed on the table, to
the right of the eard already there, and the next eard in the
staek is moved to the bottom of the staek. This proeess— plaeing
the top eard to the right of the eards already on the table and
moving the next eard in the staek to the bottom of the staek— is
repeated until all eards are on the table. It is found that, reading
left to right, the labels on the eards are now in aseending order:
1, 2, 3, . . . , 1999, 2000. In the original staek of eards, how many
eards were above the eard labeled 1999?
eards have been moved from the top of the staek to the bottom, a
2000-eard staek remains. Remove the eards that are on the table.
The next-to-last eard that will be plaeed on the table from the
2000-eard staek is the eard that began at position 1024 in the
2048-eard staek. The position of that eard in the 2000-eard staek
is 1024 — (48 + 48) = 928, so the number of eards above it is 927.
13. Form a 2000 x 2002 sereen with unit sereens. Initially, there are
more than 1999 x 2001 unit sereens whieh are on. In any 2 x 2
sereen, as soon as there are 3 unit sereens whieh are off, the 4th
sereen turns off automatieally. Prove that the whole sereen ean
never be totally off.
2n + 1
n
as desired.
( 2 ” „+ 1 >
t T H u r f i i )
V^2 n ~k \ _ f'2n + Л
t , \ k J \ [ ( n - k ) l 2 \J \ n ) ’
as desired.
16. [Bay Area Math Çirele 1999] Let m and n be positive integers.
Suppose that a given reetangle ean be tiled by a eombination of
horizontal 1 x m strips and vertical n x 1 strips. Prove that it
ean be tiled using only one of the two types.
Solutions to Advanced Problems 77
e - i
c e ( i + ^ + ç 2 + - - - + r _1) = c e = 0 .
€ - 1
Likewise the sum of the numbers in any horizontal strip is also
0. Sinee the reetangle is tiled by these strips, the sum of all the
numbers in the reetangle is 0. But this sum is equal to
C“ - 1 У - 1
(C + C' + '-- + C“)K + r + - - + П =
C -i e- i
Therefore we must have (/a = 1 or £b = 1, implying m |a or n \ b,
respectively.
17. Given an initial sequence ai, « 2 , •••, «n of real numbers, we
perform a series of steps. At eaeh step, we replaee the eurrent
sequence x\, xş, •••, x n with \x\ — a\, \x^ — a\, . . . , \xn — a\ for
some a. For eaeh step, the value of a ean be dilferent.
(a) Prove that it is always possible to obtain the null sequence
eonsisting of all 0 ’s.
(b) Determine with proof the minimumnumber of steps required,
regardless of initial sequence, to obtain the null sequence.
,(!) (i)
,(2 ) _ J 2) , ( 2) lai —аз
«1 = «2 = °3 =
78 Solutions to Advanced Problems
and so on. At the kth step, we take a(fe) = —----- »afc+1— to obtain
(fe-i) (fe-i
(.k) (fe) (fc) 4 - öfe+i
a\ ' = a\ — = afc+i =
m ^ -1) = т Ь { а ^ - 1}, a ^ , . . . , a ^ 1) }
and
We ean set the value of a at the kth step to a(fe) + a (fe+1) to save a
step, whieh eontradiets the faet that m is the minimum number
of steps needed. On the other hand, if a(fe) > M(fe х), then for
Solutions to Advanced Problems 79
all i,
• • (k) • •
whieh eontradiets the faet that — 0 as it is part of the null
sequence. Therefore our assumption was wrong and we need at
least k + 1 steps to turn the sequence 1 , 2 ! , . . . , (/ ? + 1 )! into the
null sequence. Our induetion is thus eomplete.
18. [Çhina 2000, Yuming Huang] The sequence {a „ }„ > i satisfies the
eonditions ai — 0 , 02 — 1 ,
80 Solutions to Advanced Problems
an —2
an = nan_i + ( - 1 )",
an — n
or
f n = n\ — 1 + n((n — 1 )! — 1 ) = 2 •n\ — n — 1
marks.
(n - l ) ( a „ _ i + a „ _ 2).
Solutions to Advanced Problems 81
as desired.
20. In a forest eaeh of 9 animals lives in its own cave, and there
is exactly one separate path between any two of these caves.
Before the eleetion for Forest Gump, King of the Forest, some of
the animals make an eleetion eampaign. Eaeh eampaign-making
animal— TQC (Forest Gump eandidate)— visits eaeh of the other
caves exactly onee, uses only the paths for moving from cave to
cave, never turns from one path to another between the caves,
and returns to its own cave at the end of the eampaign. It is also
known that no path between two caves is used by more than one
TQC. Find the maximum possible number of T Q C s.
Р к - 1 , Р к + 3 , Рк, Рк + 2, Рк + 1, P q)-
D-niAi, A 2 , . . . , A n) = { i G S |i ф A n^ } .
D no (A \ , A 2, . . . , A n) = 0
and
(2 , 0 , 2 , 0 ), ( 1 , 2 , 1 , 0 ), (2 , 1 , 2 , 0 , 0 ), (p, 2 , 1 , 0 _ _ A 1 , 0 , 0 , 0 ),
p—3
for p > 3.
(xq,X\, . . . , x n, n + 1 , n + 1 , . .. , n + 1 ,
4---------------v '
xn+i —n+ 1 terms
n 2, n 2 , . . . ,n-\- 2 , . . .).
4---------------V---------------"
x n + 2 terms
86 Solutions to Advanced Problems
For example,
( 1, 2 , 1, 0 , 4 , 4 , 4 , 4 , 5 , 5 , 5 , 5 , 6 , 6 , 6 , 6 , 7 , 7 , 7 , 7 , 8 , 8 , 8 , 8 , 8 , . . . ) .
24. Determine if it is possible to partition the set of positive integers
into sets Л and B sueh that A does not eontain any 3-element
arithmetie sequence and B does not eontain any infinite arith-
metie sequence.
{ ( 1 , 1) ; ( 1, 2 ) , ( 2 , 1) ; ( 1 , 3) , ( 2 , 2 ) , ( 3 , 1);
We build set A inductively. At step 1 , we put a\ = 1 in set A
(this breaks the infinite arithmetie sequence ( 1 , 1 )); at step 2 , we
piek a number larger than 2 ai = 2 from the sequence ( 1 , 2 )
and put it in A (this breaks the sequence ( 1 , 2 )); at step 3, we
piek a number that is larger than 2 a 2 from the sequence ( 2 , 1 )
and put it in A ; . . . ; at the ith step, i > 3, we piek a number that
is larger than 2 а г-_1 from the ith sequence from S (note that S is
eountable so sueh an ordering exists) and put it in A , and so on.
All the numbers that are not in A form set B.
By this eonstruetion, it is elear that every infinite sequence has
been broken, so set B eontains no infinite arithmetie sequence.
On the other hand, the elements in A ean arranged in inereasing
order а ! , а 2 , аз, . . . with аг+ 1 > 2аг. It follows that any three
terms ai < aj < eannot form an arithmetie sequence as
2 aj < aj + 1 < а^ < ak + аг.
Therefore it is possible to partition the set of positive integers
into sets A and B sueh that A does not eontain any 3-element
arithmetie sequence and B does not eontain any infinite arith-
metie sequence.
25. [USSR 1989] Çonsider the set T5 of 5-digit positive integers whose
deeimal representations are permutations of the digits 1, 2, 3, 4,
Solutions to Advanced Problems 87
5 5
О г’_|_1 О г’_|_2Ог'_|_зОг'_|_4) — ( 0 j ’_| _40j_|_30j’_|_20j_|_i f l j ) ,
i= 1 i=1
(l + x4 + xs + x 12) . . . ,
x4 — 1 x8 — 1 x 16 — 1 x 64 — 1
X —1 X2 — 1 X4 — 1 X8 — 1
1
(x — l ) ( x 2 — 1 ) ’
as eaeh term in the numerator oeeurs in the denominator of the
fraetion two terms away. By partial fraetions, we obtain
1 1 1
f { x ) = 4(x + 1) 4(x - 1) 2(x - l ) 2
1
~ 2 + 1 = -l ( ( x - l ) - 2
4(x2 — 1) 2(x — l ) 2 2 \ ; 1 x*
Expanding the two funetions in the last equation, we hnd that
№ = \ 1 - Ix + x 2 —■
+ (l + x 2 + x 4 + ■■•)
Sinee
- 2 \ _ (—2 ) (—3) •••( —2 — n + 1 )
= ( - ! ) " ( « + 1),
n n\
Solutions to Advanced Problems 89
we obtain
= Ž ( [ Ş j + 1K
m—0
Thus, the eoeffieient of x n is [n/2j +1, that is, there are [n/2j +1
polynomials satisfying the eonditions of the problem.
A B e
D E F
G H I
1 1 0 1 1 0
0 1 1 0 1
1 0 11
1 1 0
0 1
1
Now we are ready to prove our main result. Let v\, г?2 , . . ., v \ q
be the vertices of our 2 -eolored (red and blue) eomplete graph
K\q. By Lemma 1, there is a monoehromatie triangle in
K\q. Without loss of generality, say it is v i v 2vq. By Lemma
1 again, there is a monoehromatie triangle in the subgraph
K\q —{ v i , У2 , У з } . Without loss of generality, assume it is ^4 ^5 vq.
If г>1г>2гз and ^4 ^51^6 are of the same eolor, we are done. If not,
assume that г ^ г з is blue and г^г^гб is red. Çonsider the edges
vi vj i 1 £ г ^ 3 and 4 < j < 6 . By the Pigeonhole Prineiple, ňve
of them are of the same eolor; without loss of generality, assume
they are blue. Henee there is some j 0 , 4 < j 0 < 6 , sueh that two
of the edges Vj0vi, Vj0V2 , Vj0vs are blue. Therefore we have one
blue triangle and a red triangle with exactly one eommon vertex
V3o■
For simplieity, we relabel the points so that г ^ г з is blue and
^3 ^4^5 is red. Çonsider the subgraph K \ q — {г^, v2, ■■■, vs }. If
it has a monoehromatie triangle, we are done as we ean piek one
of the triangles v\v2vs and гзг 4^5 to mateh the eolor of this new
triangle. Therefore one airline ean provide two traveling eyeles of
three eities with no eities in eommon. If not, by Lemma 2, we have
a red length-5 eyele and a blue length-5 eyele. Therefore eaeh
airline ean provide a traveling eyele of three eities and traveling
eyele of five eities with no eommon eities.
31. [MOSP 1997] Suppose that eaeh positive integer not greater than
n (n 2 — 2n + 3)/ 2 , n > 2 , is eolored one of two eolors (red
or blue). Show that there must be a monoehromatie n-term
sequence ai < a2 < •••< an satisfying
We shall prove that if the integers are eolored red and blue, the
first sn integers eontain a monoehromatie n-term 3 ( 2)-sequence.
Solutions to Advanced Problems 95
®n+ l $n
n n
+
< ” ‘) y t v c ; r . 1 ,
■ Ч У Ч + '
Çonsider the list of n + 1 numbers
n n
+ 1, . . ., an + 3 n
< Sn + З Г + r j + 1 = s„+1.
^n+ l Cln — 3 - * ( + ‘ s + r
If x E C , then x — 1 6 A. ( 1)
Then a eontradietion will follow immediately from (1). Indeed,
if C — { e i , С2 , . . ., cn}, then A eontains the numbers c\ — l , e^ —
96 Solutions to Advanced Problems
10. Henee
— У м ) = 10.
м
Note that for eaeh M , х м — Ум < ^м < 1- Therefore there are
at least 11 terms in the above summation that are positive, i.e.,
there are at least 11 MOPpers M with х м > У м , whieh is what
we wanted to prove.
34. [USAMO 1999 submission, Bjorn Poonen] Starting from a triple
(a, b, e) of nonnegative integers, a move eonsists of ehoosing two
of them, say x and y, and replaeing one of them by either x + y
or \x — y |. For example, one ean go from (3, 5, 7) to (3, 5, 4)
in one move. Prove that there exists a eonstant r > 0 sueh that
whenever a , b , c , n are positive integers with a,b ,c < 2n, there is a
sequence of at most rn moves transforming (a, b, e) into (a', b', e')
with a'b'c' = 0.
where 0 < e < b, i\ < U < •••< ii, and x^t < y < Xit+i.
Proof: Sinee х^ are inereasing, there is a unique i > 1 for
whieh Xi < y < х^+1 . We use strong induetion on i. If y —х^ < b,
we let e = y — Xi and we are done. Otherwise x\ = b < y — X{ <
Xi+i — х^ = X i- i. Thus there is a unique j > 1 sueh that
х^ < y — Xi < Xj +1 , and j < i, so we hnish by applying the
inductive hypothesis to y — X{. u
Write e = e + +'i + •••+ Xit , where 0 < e < b and 0 < ii <
•* •< ii. Sinee x k+2 = x k+i + x k = 2xk + x k~i > 2xk for k > 1,
we have
distinet.
(ii) Step 2: If m = 1 we may skip this step. Otherwise,
reeall that by Euler’s extension of Fermafs Little Theorem,
2 ф(т) = ү (mod m). Let r = ф{т). Now we add to T the
elements 2 S t , T T t , . . . , 2r_1E T , as well as (2r —1) Et , 2(2r —
1)ST , 4(2r - 1)ET , .. •, 2r_2(2r - 1)ET . The new set U thus
formed has sum E u = 2r_1(2r — 1)ET - Thus all of the
elements we added divide E u, as do all of the elements of
T that divided E T - Furthermore, m |E j j ■
After performing these two steps, we have s |S u ■Furthermore,
all of the elements that originally divided E s still divide E[/, and
all of the elements that we added also divide E jj ■ Henee if we
had n elements of S whieh did not divide E s, we now have at
most n — 1 elements of T whieh do not divide E T .
Thus, if we ean eonstruet a DS-set eontaining any subset with
n elements not dividing the sum, then we ean eonstruet a DS-set
eontaining as a subset any set with n + 1 elements not dividing
the sum. By induetion, we are done.
= (n + 1)!,
sinee the sum teleseopes. It is elear that the elements of our set
are distinet and divide (n + 1)!, so the proof is eomplete.
Solutions to Advanced Problems 101
f e —i if i < e — s, .
Pi = 1f e — z +. ±l 1 rii г :> e — s. i
satisfy
Qi - q j = P i — 0 > P i - P k + 2 > ( i — k — l) + 2 = i —j - l .
« 5 , so П5 = 11748.
n5 + n6, so uq — 116430.
(l + 8 5 (3 ) ” 5 + (5) n6 + ( l ) n 7, so n7 1806954f,
П П
^ J ik + 'У J jk — 1 + 2 + •••+ 2n — n(2n + 1) — S i ,
k =1 k=1
- £ . • * = 2 + 3 + - . + (n + l) = ! ^ ± ® ) = S 2l
k=1 k=1
2m — 1; (4m — 3, 4m — 5, . . . , 2m + 1); 4m — 2;
(2m — 2, 2m — 4 , . . . , 2); 2m — 1; 4m — 1;
2m — 1; (4m — 3, 4m — 5, . . . , 2m + 1); 4m — 1;
{1,2,...,24}
into k pairs of numbers (ai, bi), (ü 2 , b^), •••, (ak , bk) sueh that
bi — а^ = i + 1 for 1 < i < k.
• Skolem Determine if it is possible to partition the set
{ 1, 2 , . . . , 2 4 }
into k pairs of numbers (ai, bi), (a^, b^), . . ., (ak , bk) sueh that
bi — щ = i for 1 < i < k.
• Skolem Determine if it is possible to partition the set
{2,3,...,24}
43. [IMO Shortlist 1996] Let k , m , n be integers sueh that 1 < n <
m —1 < k. Determine the maximum size of a subset S of the set
{1, 2 , . . . , k } sueh that no n distinet elements of S add up to m.
+ (r + 1) + h (r + n - 1) < m,
then
S C { r + 1, r + 2, . . . , k }
n — l < m — nx — 1 < k — x .
m —x n
= k
n —1 2
mn — nx 1 n —1
= k—
n(n — 1) 2 2
The inequality nx + n(n — l ) / 2 < m implies that mn — nx >
(n — 1)m + n(n — 1)/ 2 . It follows that
m n —1
n n+ 1
C(A) < <
n+ 1 n
If the numbers in G are in different rows and eolumns, then two
of them are on the same row or eolumn as the number n2 —n, so
we have
n —1 n+ 1
C(A ) <
n —n n
Now we show that the arrangement
i + n(j 1) if i < j ,
aij —
i + n(n — i + j — 1) if i > j ,
114 Solutions to Advanced Problems
that is,
1 + (n — 1)n 1 1 + (n — 2 )n
2 + (n — 2 )n 2 + (n — 1)n 2 + (n — 3)n
eo
eo
eo
3 + (n — 2) n
1
+
+
(n — 2) + 2 n (n — 2) + 3n (n — 2) + n
(n — 1) + n (гг — 1) + n —1
n n+n n + (n — l) n
implying
ац 1 + (n - l)n n —n + 1 n+ 1
aki 2 + (n — 2 )n n2 — 2n + 2 n
with equality if n = 2;
• The j th eolumn, 2 < j < n, eontains the two arithmetie
progressions:
j ~ 1, (j ~ 2) + n, ( j - 3) + 2 n , . . .,1 + (j - 2)n;
« + (j - l)ra, (n - 1) + j n , . . ., j + (n - l)ra,
implying
aij j + ( n - 1) n n+ 1
> >
akj ~ (j + 1) + (n — 2)tt n
with equality for j = n — 1.
46. [Çhina 1999, Hongbin Yu] For a set 51, let |5| denote the number of
elements in S. Let A be a set with |ff| = n, and let A\, A 2 , .. .,
be subsets of A with \Ai \ > 2, 1 < i < n. Suppose that for eaeh
116 Solutions to Advanced Problems
V li; = V V —
ds
> V У'
1-4,, 4 ,
^ ^ n —di ~ ^ ^ n — \Aj
i= 1 i= lj\ x ig A j i= lj\ x ig A j
1-4,
П- |Aj|
= E i m
j =i
By (2), all the equalities hold in the above inequalities. Henee
dj — |Aj|. It follows that
П П П
- di)dj = Y di = Y Y
i —1 i —1 j | *= 1 j |
n
=£ lAil
i =l i |
n
= E ( " - i^ di ^ i
i=i
implying (3), as desired.
47. [Iran 1999] Suppose that r\, . . ., rn are real numbers. Prove that
there exists a set S C {1, 2, .. ., n) sueh that
1
Y n r,: .
i£S i=1
Sj — rj
'J and tj = ri-
r j T 0 , J = 1 ( mo d 3) r i < 0 j = * ( m o d 3)
2 s — ( §i + § 2 ) + ( s 2 + 5з) + ( s 3 + s l )
— ( fl + ^2 ) — (f 2 + *з) — (<3 + f l ) -
Solutions to Advanced Problems 117
m n m
\S\ = n k \k + 1 / k + 1 \k,
^ n \ m{m — l ) { n — k){n — k + 1 )
m(m — 1 )
ф + l) k{k + 1 )
On the other hand, we ean also sort the triples by V(1W , whieh is
always a set of k — 1 elements. For eaeh (k —l)-element subset J
of { 1 , 2 , . . ., n ) , let s j be the number of elements of S eontaining
it. Eaeh J is V П W for exactly s j { s j —1) triples, so we eonelude
Henee
m(m — l)(n — k)(n — k -\- 1 )
*(* + 1)
m2(n — k + l ) 2 m(n —k + 1 )
- (^ + l ) 2 &+ 1
or
(m — 1 )(n — &) m(n —k + 1 )
k ~ k+ 1
Henee
A A B = (A - B ) U (B - Ä)
S = { S u S 2 , . . . , S 2m}.
S h A S i2A - - - A S i 2j
Ti = A S 2A ■■■A S k A S k + i A ■■■A S 2i
= S k+1A ■- . A S n A S w A S i j - k = T 2
0 = Ti A T2
S 1A S 2 - - - A S 2i = 0 or S.
S i A S 2 ■■■A S 2i = 0.
S 1A S 2 - - - A S 2i = S.
Solutions to Advanced Problems 121
2i
Therefore
x£X x £ ( X —i)
= a{ -
х £ ( X —i)
(b) В 2 П- 1 — A 2n
(c) В 2 П = 2 B n ;
(d) 1 E B^n-i-
The base ease ean be verified by eomputing A 2 = { 1 } , B 2 =
{0}, = {1}, B 3 = { 0 , 1 } , H4 = { 1 , 2 } , and B 4 = { 0 } .
For the induetion step, assume that the statement holds for
n — 1; that is, assume that А 2 П- з = 2An-\ — 1, В^п-з =
Solutions to Advanced Problems 123
= 2(An - l) + l = 2An - l ,
= (2Bn- i + 1) U 2B n ,
portion. Let n = 2m. We must show that for all 0 < k < 2m_1,
/ 2 m - 1 - k \ _ (2m — 1 — Z)(2m — 2 — k) ■■■(2m — 2 /)
V / У Ar ■ ( ät — 1 ) - - - 1
n — i — 2m = —i (mod 2m)
for all i. Thus, for all i < 2m, the faetor n — i — 2m in the
numerator has exactly the same number of faetors of 2 as the
faetor i in the denominator. The same is true for the last pair
of faetors, n — 2m+1 and 2m. Neither is divisible by 2m+1 while
-•ya 1 T \
both are divisible by 2m, so the binomial eoeffieient ( ~2~ ) is
indeed odd.
51. [Iran 1999] Suppose that S = {1, 2, . . ., n } and that A \ , A 2, . . .,
А^ are subsets of S sueh that for every 1 < * i< 2 < 3 < 4 < k, we
have
|Л\ U 4 u 4 u4|<n-2.
126 Solutions to Advanced Problems
P ro v e t h a t k < 2n~2.
\At U A v U A{ U A j |= n - 1,